1989 AHSME Problems/Problem 16

Revision as of 22:17, 29 February 2012 by Ckorr2003 (talk | contribs) (Created page with "Since the endpoints are (3,17) and (48,281), the line that passes through these 2 points has slope <math>m=\frac{281-17}{48-3}=\frac{264}{45}=\frac{88}{15}</math>. The equation o...")
(diff) ← Older revision | Latest revision (diff) | Newer revision → (diff)

Since the endpoints are (3,17) and (48,281), the line that passes through these 2 points has slope $m=\frac{281-17}{48-3}=\frac{264}{45}=\frac{88}{15}$. The equation of the line passing through these points can then be given by $y=17+\frac{88}{15}(x-3)$. Since $\frac{88}{15}$ is reduced to lowest terms, in order for $y$ to be integral we must have that $15|x-3$. Hence $x$ is 3 more than a multiple of 15. Note that $x=3$ corresponds to the endpoint $(3,17)$. Then we have $x=18$, $x=33$, and $x=48$ where $x=48$ corresponds to the endpoint $(48,281)$. Hence there are 4 in all.